On the other hand, we can write
4.3 GENERATING FUNCTIONS 141
U(x) = (1 +x) (1 +x^2 )(1 +x^3 )(1 +x^4 )(^1 +x^5 ) ...
=
(\�:) C =�:) C =�:) (� =�) Cl �:50 ) ....
Notice that in this last expression, we can cancel out all the terms of the form (1_�k),
leaving only
1
- ( l---x-) (-I ---x 3 =-)-( 1 - -x-=-^5 )-( 1--x-=7-) .-..
= V (x). •
Problems and Exercises
4.3.8 Prove that for any positive integer n,
4.3.9 Prove that for any positive integers k < m, n,
�
(
�
) (
m
.) = (
n+m
)
f;:o.
J k-J k
4.3. 10 Use generating functions to prove the formula
for the Fibonacci series given in Problem 1.3.18 on
page 10.
4.3. 11 Reread Example 4.3.6. Prove formally by in
duction that if
then
Jr (x) := (I +x+[^1 )(1 +x^3 +x-^3 ) x
(I +�2 +[^32 ) ... (I +�' +x-^3 '),
Jr (x) = x" +x"-I + ... +x^2 +x+ I
+ x-I +x-^2 +. .. +x-a,
where a = (3'+^1 - 1)/2.
4.3. 12 Here is an alternative way to end Exam
ple 4.3.7: Show that F ( x) is equal to I for all x, where
F(x) := (l-x)(1 +x) x
(I-�)(I +�) x
(I _x^5 )(1 +�) x
(I-x7)(1 +x^4 ) x
Here are suggestions for two different arguments:
I. Try induction by looking at the expansion of the
first 2n terms of F(x). It won't equal I, but it
should equal something that does not have any
xl< terms for "small" k. The idea is to show that
as n grows, you can guarantee that there are no
xl< terms for k = 1, 2 , ... ,L, where L is some
thing that grows as n grows. That does it!
- Show that F(x) is invariant under the substitu
tion x >-> x^2 • Then keep iterating this substitu
tion (you may also want to note that the expres
sions only converge when Ixl < I).
4.3.13 (Putnam 1992) For nonnegative integers n and
k, define Q(n,k) to be the coefficient of xl< in the ex
pansion of (I +x+x^2 +�)n. Prove that
Q(n,k) =
± (
�
) (k �2 .)
.
} =o J J
4.3. 14 Show that every positive integer has a unique
binary (base-2) representation. For example, 6 is rep
resented by 110 in binary, since I· 2^2 + I. 21 +0. 20 =
- (This uniqueness can be proven in several ways; you
are urged to try generating functions here, of course.)
4.3. 15 The Root oj Unity Filter. Let S = Cis^2 : be an
nth root of unity (see page 1 26).
(a) Show that the sum
1 + Sk + S2k + S^3 k + ... + S(n-I)k
equals n or 0, according to whether k is a multi
ple of n or not.
In/ (^2) J
( )
(b) Find a simple formula for the sum }: ;..
}=o J